Thursday, October 28, 2010

Chris - 10/27

5/3/1, Wave A, Bench

Bench Press:
Warmup: 45x5; 65x5; 85x5; 95x3
105x5
120x5
140x14
I blanked out a bit between 7 and 8 so I either did 14 or 15. Theoretical max 205-210 depending on what I actually did.

Bench Press (assistance):
85x10; 85x10; 85x10; 85x10; 85x10
So much bench volume...

Dumbbell Rows (reps are per hand):
65x15; 65x16; 65x20
I picked up the closest dumbbell and started rowing it.

I woke up today with the worst DOMS I've had in a long, long time. I don't really expect anything different tomorrow. This is gonna be fuuuuun.

7 comments:

  1. I don't think that predicting / estimating / whatever 1RM based on more than 5 reps makes sense, and even then it's just an educated guess. For instance, as a result of a friendly bet I squatted last week 180x20; what is my theoretical max?

    ReplyDelete
  2. Of course it wouldn't make sense if I were to suddenly test a 1RM in the middle of my cycle to "confirm" whatever number I think is my max. Rather, this is just a benchmark number; so long as I see my "theoretical max" rise across several cycles of 5/3/1 (or whatever program I'm on), I can be somewhat secure in the fact that I'm making some progress; I'm clearly going to be stronger if next time I squat 300x10 instead of 295x10.

    But you're right, listing it as a "theoretical max" isn't constructive. I can't think of any other decent names for it though, other than "Jim Wendler's Arbitrary Plug'n'Play Progress Number."

    ReplyDelete
  3. Just for reference, 180kg x 20 reps predicts a 1rm of 300 kg.

    ReplyDelete
  4. (Weight/30)*reps + weight.
    63% = 20 RM
    75% = 10 RM
    90% = 3RM
    95% = 2RM

    just handy little approximations that you can use as guidelines. (In general, not simply pertaining to 531).

    ReplyDelete
  5. I know what you mean about DOMS, my butt is still sore.

    ReplyDelete
  6. 5/3/1 is no joke- my thighs and buttocks are sore as hell... btw Chris what assistance exercise are you choosing for the military press day?

    ReplyDelete